====== Paracompacidade ====== === Definição: Localmente finito === Seja $\mathcal{F}$ família de subconjuntos de $X$. Dizemos que $\mathcal{F}$ é **localmente finita** se $\forall x \in X$ , $\exists$ $V$ aberto tal que $x \in V$ e {$F \in \mathcal{F} \ | \ F \cap V = \emptyset$} é finito. === Definição: Refinamento === Seja $\mathcal{C}$ cobertura de $X$. Dizemos que $\mathcal{C}'$ é um **refinamento** para $\mathcal{C}$ se $\mathcal{C}'$ é cobertura e se $\forall A \in \mathcal{C}'$, $\exists$ $C \in \mathcal{C}$ tal que $A \subset C$. === Definição: Paracompacto === Dizemos que $X$ é **paracompacto** se toda cobertura admite refinamento aberto localmente finito. === Exemplos === - Todo espaço compacto é paracompacto. - A reta de Sorgenfrey é paracompacta (mesmo não sendo compacta), mas o plano de Sorgenfrey não. - Todo espaço de Lindelöf regular é paracompacto. === Exercícios === * Seja $(X, \tau)$ espaço topológico $T_1$. Mostre que $D \subset X$ é discreto com a topologia induzida e fechado $\Leftrightarrow$ {$ \{d\} \ | \ d \in D$} é localmente finito. * Seja $(X, \tau)$ um espaço topológico com base enumerável e regular. Então $(X, \tau)$ é paracompacto. * tome $\mathcal{B}$ base enumerável para $X$ e para cada $x \in X$ pegue $B_x, C_x \in \mathcal{B}$ tais que $x \in B_x \subset \overline{B_x} \subset C_x \subset C$, com $C \in \mathcal{C}$; * fixe $(x_n)_{n \in \mathbb{N}}$ de forma que {$B_{x_n} \ | \ n \in \mathbb{N}$} = {$B_x \ | \ x \in X$}; * defina $W_0 = C_{x_0}$ e $W_n = C_{x_n} - (\cup_{k < n} \overline{B_{x_k}})$ e prove que os $W_n$ formam uma cobertura, são um refinamento e também localmente finitos. * Seja $\mathcal{F}$ uma família localmente finita. Então, $\cup_{F \in \mathcal{F}} \overline{F} = \overline{\cup_{F \in \mathcal{F}} F}$. Em particular, $\cup_{F \in \mathcal{F}} \overline{F}$ é fechado. * sabemos que $\cup_{F \in \mathcal{F}} \overline{F} \subset \overline{\cup_{F \in \mathcal{F}} F}$; * para provar a inclusão contrária, tome $x \in \overline{\cup_{F \in \mathcal{F}} F}$ e $A$ um aberto de forma que o conjunto $\mathcal{F}_0 = \{ F \in \mathcal{F} \ | \ F \cap A \neq \emptyset \}$ é finito; * note que $x \in \overline{\cup_{F \in \mathcal{F}_0} F}$ e use o fato de $\mathcal{F}_0$ ser finito para concluir o resultado. * Sejam $(X, \tau)$ um espaço paracompacto e $A, B \subset X$ fechados disjuntos. Se $\forall X \in B$, existem abertos disjuntos $U_x$ e $V_x$ tais que $A \subset U_x$, $ x \in V_x$, então existem $U, V$ abertos disjuntos de forma que $A \subset U$ e $B \subset V$. *Sejam $(X, \tau)$ um espaço paracompacto e $A, B \subset X$ fechados disjuntos. Se $\forall X \in B$, existem abertos disjuntos $U_x$ e $V_x$ tais que $A \subset U_x$, $ x \in V_x$, então existem $U, V$ abertos disjuntos de forma que $A \subset U$ e $B \subset V$. * tome a cobertura aberta $\{ V_x | x \in B \} \cup \{ X - B \}$; * como $X$ é paracompacto, existe refinamento aberto e finito $\{ W_s \ | \ s \in S \}$; * note que $\overline{W}_s \cap A = \emptyset$ (para algum $x$); * faça $V = \cup_{s \in S} W_s$ e note que, pela definição do refinamento, $B \subset V$; * use o exercício anterior pra provar que $\cup_{s \in S} \overline{W}_s$ é fechado e tome $U$ como o sendo seu complementar. * Todo espaço de Hausdorff paracompacto é normal. * pelo lema anterior, tome $A = \{x \}$ e $B$ fechado tal que $x \notin B$ e veja que $X$ é regular; * novamente aplicando o lema, por $X$ ser regular, segue que $X$ é normal. * Todo espaço métrico é paracompacto. * comece tomando $(X, d)$ um espaço métrico, $\mathcal{C}$ uma cobertura aberta para $X$ e $\preceq$ uma boa ordem sobre $\mathcal{C}$; * para cada $C \in \mathcal{C}$, defina a família de conjuntos $\left(D_{n}(C)\right)_{n \in \mathbb{N}_{>0}}$ utilizando indução sobre $n$, da seguinte forma * $D_{1}(C)=\bigcup_{x \in A} B_{\frac{1}{2}}(x)$, com $A=\left\{x \in C \ | \ C=\min \left\{C^{\prime} \in \mathcal{C}: x \in C^{\prime}\right\} \text { e } B_{\frac{3}{2}}(x) \subset C\right\}$; * $D_{n}(C)=$ $\bigcup_{x \in A} B_{\frac{1}{2^{n}}}(x)$, sendo $A= \left\{x \in C \ | \ C=\min \{C^{\prime} \in \mathcal{C} \ | \ x \in C^{\prime} \}, \ x \notin D_{k}(C^{\prime}) \ \text{para} \ \text{qualquer} \ k0}, C \in \mathcal{C}\right\}$ seguindo os seguintes passos: * note que $D_{n}(C) \subset C$; * para ver que de fato é cobertura, tome $x \in X$ e $C=\min \left\{C^{\prime} \in \mathcal{C}: x \in C^{\prime}\right\}$; * perceba que existe algum $n \in \mathbb{N}$ de forma que $B_{\frac{3}{2^{n}}}(x) \subset C$ $\Rightarrow$ $x \in D_{n}(C)$ ou $x \in D_{k}\left(C^{\prime}\right)$ para algum $k \leq n$ e $C^{\prime} \in \mathcal{C}$; * agora, para $\left\{D_{n}(C): n \in \mathbb{N}_{>0}, C \in \mathcal{C}\right\}$ é localmente finito, tome $x \in X$ e $C=\min \left\{C^{\prime} \in \mathcal{C}: x \in D_{n}\left(C^{\prime}\right)\right.$ para algum $\left.n \in \mathbb{N}\right\}$. Tome também $j \in \mathbb{N}$ de forma que $B_{\frac{1}{2 j}}(x) \subset D_{n}(C)$, onde $n$ é tal que $x \in D_{n}(C)$. Daí, mostre que: * Se $i \geq n+j$, então $B_{\frac{1}{2^{n+j}}}(x)$ não intercepta $D_{i}\left(C^{\prime}\right)$ para qualquer $C^{\prime} \in \mathcal{C}$; * Se $i